Strengthen with Necessary Premise Questions - - Question 25

The reforms to improve the quality of public education that have been initiated on the part of suppliers of public ed...

TheFacu June 11, 2015

Explain

Please explain a-e

Replies
Create a free account to read and take part in forum discussions.

Already have an account? log in

Naz June 12, 2015

Here we have a strengthen with necessary premise question. Remember that a premise is necessary for a conclusion if the falsity of the premise guarantees or brings about the falsity of the conclusion. First we check to see if the answer choice strengthens the passage, and then, if it does strengthen, we negate the answer choice to see if its negation makes the argument fall apart. If the answer choice does both those things then it is our correct answer.

Conclusion: "Therefore, reforms must be demanded by consumers."

Premise: The reforms to improve the quality of public education that have been initiated on the part of suppliers of public education have been insufficient. We are told that parents should be given government vouchers with which to pay for their children's education and they should be allowed to choose the schools at which the vouchers will be spent. And finally, we are told that in order to attract students, academically underachieving schools will be forced to improve their academic offerings.

So, our subsidiary conclusion here is that underachieving schools will have to improve academically in order to attract students, whose parents will be spending money with government paid vouchers.

Well, what's the issue here? We've never discussed that what makes a school attractive for parents is its academic success. If school location or faculty to student ratio was what made a school attractive, then academically underachieving schools may not be forced to improve their academic offerings.

So, let's look at answer choice (A): "in selecting schools parents would tend to prefer a reasonable level of academic quality to greater sports opportunities or more convenient location."

Does answer choice (A) strengthen? Yes.

Here we are solidifying the idea that a parents preference in selecting a school is academics, which would then strengthen our subsidiary conclusion that academically underachieving school would be forced to improve their academic offerings in order to be more attractive.

Negation: "in selecting schools parents would not necessarily tend to prefer a reasonable level of academic quality to greater sports opportunities or more convenient location."

Does the negation make the answer fall apart? Yes.

If academics was not preferred to other characteristics, then it no longer stands that academically underachieving schools will be forced to improve their academics.

If you have any specific questions on another answer choice, please feel free to clarify.

Hope that clears things up! Please let us know if you have any other questions.

Jay75 June 17, 2018

can you please explain why B would be incorrect?

Anita June 18, 2018

B would be incorrect because we don't know from the prompt what relationship there is, if any, between the schools and the job market. The prompt does not assume any relationship between the two, as it doesn't discuss the job market at all.

KevinRon October 30, 2018

Why is E incorrect?

Mehran October 31, 2018

Hi @KevinRon, thanks for your post. As explained above, this is a Strengthen with Necessary Premise question. In the stimulus, the conclusion is "Therefore, reforms must be demanded by consumers." The premise given is that the reforms to improve the quality of public education that have been initiated on the part of suppliers of public education have been insufficient. We are told that parents should be given government vouchers with which to pay for their children's education and they should be allowed to choose the schools at which the vouchers will be spent. And, finally, we are told that in order to attract students, academically underachieving schools will be forced to improve their academic offerings.

Thus, the subsidiary conclusion here is that underachieving schools will have to improve academically in order to attract students, whose parents will be spending money with government paid vouchers.

The problem, as explained above, is that it is never established that that which makes a school attractive for parents is its academic success.

Answer choice (E) is incorrect because it does not "bridge the gap" we have identified in the argument presented in the stimulus. Even if some schools specialized, rather than improved all their academic offerings on the whole, certain parents would send their students to those schools. Consider, for example, a school that specialized in math but did not improve its other academic offerings (e.g., in writing, foreign languages, science, or history). Parents who wanted their children to specialize in math might spend their vouchers on this school, even though it was an "academically underachieving" school in those other subjects. Accordingly, (E) does not strengthen the argument.

By contrast, answer choice (A) "bridges the gap" identified - it very explicitly says that parents would choose where to spend their money because of *academic quality* rather than some other metric (e.g., sports opportunities or more convenient location). As this is a Strengthen with Necessary premise question, the negation of this answer - "in selecting schools, parents would not necessarily tend to prefer a reasonable level of academic quality to greater sports opportunities or more convenient location" - would make the argument in the stimulus fall apart.

Hope this helps clarify, but please do let us know if you have any additional questions.